Add missing figures and solutions to Serway & Jewett v8's chapter 29.
[course.git] / latex / problems / Serway_and_Jewett_8 / problem29.02.tex
index 6c133b67068d105e2d6ef6a727f20f5036778b70..c54394d15c84cb72397bac2d606dfa318f985d47 100644 (file)
@@ -1,4 +1,4 @@
-\begin{problem*}{29.3}
+\begin{problem*}{29.2}
 Determine the initial direction of the deflection of charged particles
 as they enter the magnetic fields shown in Figure~P29.2.
 \begin{center}
@@ -32,7 +32,7 @@ Vector v = Velocity(a.center(), dir=0);
 v.draw();
 a.draw();
 
-label("\Part{a}", (0,0.5*height), N);
+label("\Part{a}", (0,-0.5*height), S);
 \end{asy}
 \hspace{\stretch{1}}
 \begin{asy}
@@ -60,7 +60,7 @@ Vector v = Velocity(a.center(), dir=180);
 v.draw();
 a.draw();
 
-label("\Part{b}", (0,0.5*height), N);
+label("\Part{b}", (0,-0.5*height), S);
 \end{asy}
 \hspace{\stretch{1}}
 \begin{asy}
@@ -89,7 +89,7 @@ Vector v = Velocity(a.center(), dir=180);
 v.draw();
 a.draw();
 
-label("\Part{c}", (0,0.5*height), N);
+label("\Part{c}", (0,-0.5*height), S);
 \end{asy}
 \hspace{\stretch{1}}
 \begin{asy}
@@ -126,11 +126,29 @@ Vector v = Velocity(a.center(), dir=90);
 v.draw();
 a.draw();
 
-label("\Part{d}", (0,0.5*height), N);
+label("\Part{d}", a.lc.center, S);
 \end{asy}
 \hspace{\stretch{1}}
+\rule{0pt}{0pt}
 \end{center}
 \end{problem*}
 
 \begin{solution}
+The force on a charged particle moving through a magnetic field is
+$\vect{F}=q\vect{v}\times\vect{B}$.  From Newton second law,
+$\vect{a}=\vect{F}/m=q/m\cdot\vect{v}\times\vect{B}$.  We can find the
+direction of deflection comes using this formula and the right hand
+rule.
+
+\Part{a}
+\ans{Up}.
+
+\Part{b}
+\ans{Out of the page}.
+
+\Part{c}
+\ans{No deflection}.
+
+\Part{d}
+\ans{Into the page}.
 \end{solution}